K
Khách

Hãy nhập câu hỏi của bạn vào đây, nếu là tài khoản VIP, bạn sẽ được ưu tiên trả lời.

trả lời:

ta có:\(\frac{1}{x}+\frac{1}{y}+\frac{1}{z}=0\)

\(\Rightarrow\hept{\begin{cases}\frac{1}{x}+\frac{1}{y}=-\frac{1}{z}\\\frac{1}{y}+\frac{1}{z}=-\frac{1}{x}\\\frac{1}{x}+\frac{1}{z}=-\frac{1}{y}\end{cases}}\)

\(Q=\frac{x+y}{z}+\frac{y+z}{x}+\frac{z+x}{y}\)

\(=\frac{x}{z}+\frac{y}{z}+\frac{y}{x}+\frac{z}{x}+\frac{z}{y}+\frac{x}{y}\)

\(=\left(\frac{x}{z}+\frac{x}{y}\right)+\left(\frac{y}{z}+\frac{y}{x}\right)+\left(\frac{z}{x}+\frac{z}{y}\right)\)

\(=x\left(\frac{1}{z}+\frac{1}{y}\right)+y\left(\frac{1}{z}+\frac{1}{x}\right)+z\left(\frac{1}{x}+\frac{1}{y}\right)\)

\(=x\left(-\frac{1}{x}\right)+y\left(-\frac{1}{y}\right)+z\left(-\frac{1}{z}\right)\)

\(=\left(-1\right)+\left(-1\right)+\left(-1\right)\)

\(=-3\)

~hok tốt~

27 tháng 11 2019

Cách ngắn hơn ạ: \(Q=\frac{x+y}{z}+\frac{y+z}{x}+\frac{z+x}{y}\)
\(=\frac{x+y}{z}+1+\frac{y+z}{x}+1+\frac{z+x}{y}+1-3\)
\(=\frac{x+y+z}{z}+\frac{x+y+z}{x}+\frac{x+y+z}{y}-3\)
\(=\left(x+y+z\right)\left(\frac{1}{x}+\frac{1}{y}+\frac{1}{z}\right)-3\)
\(=-3\)

26 tháng 11 2019

Q= -3 đúng k ạ

8 tháng 8 2018

Ta có: \(\frac{1}{x}+\frac{1}{y}+\frac{1}{z}=0\)

\(\Rightarrow\frac{1}{x}+\frac{1}{y}=-\frac{1}{z}\)

      \(\frac{1}{x}+\frac{1}{z}=-\frac{1}{y}\)

      \(\frac{1}{y}+\frac{1}{z}=-\frac{1}{x}\)

\(A=\frac{x+y}{z}+\frac{x+z}{y}+\frac{y+z}{x}=\frac{x}{z}+\frac{y}{z}+\frac{x}{y}+\frac{z}{y}+\frac{y}{x}+\frac{z}{x}\)

\(=\left(\frac{y}{z}+\frac{y}{x}\right)+\left(\frac{x}{z}+\frac{x}{y}\right)+\left(\frac{z}{y}+\frac{z}{x}\right)\)

\(=y\left(\frac{1}{z}+\frac{1}{x}\right)+x\left(\frac{1}{z}+\frac{1}{y}\right)+z\left(\frac{1}{y}+\frac{1}{x}\right)\)

\(=y.\frac{-1}{y}+x.\frac{-1}{x}+z.\frac{-1}{z}=-1-1-1=-3\)

Vậy nên A = -3

20 tháng 8 2020

\(a^2-2b+6b+b^2=-10\)

\(\Leftrightarrow a^2-2a+6b+b^2+10=0\)

\(\Leftrightarrow\left(a^2-2a+1\right)+\left(b^2+6b+9\right)=0\)

\(\Leftrightarrow\left(a-1\right)^2+\left(b+3\right)^2=0\left(1\right)\)

Vì \(\hept{\begin{cases}\left(a-1\right)^2\ge0\forall a\\\left(b+3\right)^2\ge0\forall b\end{cases}\Leftrightarrow\hept{\begin{cases}\left(a-1\right)^2=0\\\left(b+3\right)^2=0\end{cases}\Leftrightarrow}\hept{\begin{cases}a=1\\b=-3\end{cases}}}\)

\(L=\frac{x+y}{z}+1+\frac{y+z}{x}+1+\frac{x+z}{y}+1-3\)

\(=\left(x+y+z\right)\left(\frac{1}{x}+\frac{1}{y}+\frac{1}{z}\right)-3=0-3=-3\)

17 tháng 11 2016

\(x^3+y^3+z^3=3xyz\)

\(x^3+y^3+z^3-3xyz=0\)

\(\left(x+y+z\right)\left(x^2+y^2+z^2-xy-xz-yz\right)=0\)

\(x^2+y^2+z^2-xy-xz-yz=0\left(x+y+z\ne0\right)\)

\(2\times\left(x^2+y^2+z^2-xy-xz-yz\right)=0\times2\)

\(2x^2+2y^2+2z^2-2xy-2xz-2yz=0\)

\(x^2-2xy+y^2+x^2-2xz+z^2+y^2-2yz+z^2=0\)

\(\left(x-y\right)^2+\left(x-z\right)^2+\left(y-z\right)^2=0\)

\(\left[\begin{array}{nghiempt}x-y=0\\x-z=0\\y-z=0\end{array}\right.\)

\(\left[\begin{array}{nghiempt}x=y\\x=z\\y=z\end{array}\right.\)

x = y = z

\(P=\left(1+\frac{x}{y}\right)\left(1+\frac{y}{z}\right)\left(1+\frac{x}{z}\right)\)

\(=\left(1+\frac{x}{x}\right)\left(1+\frac{y}{y}\right)\left(1+\frac{z}{z}\right)\)

\(=\left(1+1\right)\left(1+1\right)\left(1+1\right)\)

\(=2^3\)

\(=8\)

1 tháng 3 2017

Làm sao để ra được dòng thứ 3 ak??

27 tháng 12 2016

Câu trả lời là thiếu dự kiện

10 tháng 7 2016

\(A=\frac{x+y}{z}+1+\frac{x+z}{y}+1+\frac{y+z}{x}+1-3\)

\(A=\frac{x+y+z}{z}+\frac{x+y+z}{y}+\frac{x+y+z}{x}-3\)

\(A=\left(x+y+z\right)\cdot\left(\frac{1}{x}+\frac{1}{y}+\frac{1}{z}\right)-3=\left(z+y+z\right)\cdot0-3=-3\)

Vậy, A = -3

10 tháng 7 2016

cảm ơn bạn nha

17 tháng 12 2018

Ta có: \(\frac{1}{x}+\frac{1}{y}+\frac{1}{z}=0\Leftrightarrow\hept{\begin{cases}\frac{1}{x}+\frac{1}{y}=-\frac{1}{z}\\\frac{1}{y}+\frac{1}{z}=-\frac{1}{x}\\\frac{1}{x}+\frac{1}{z}=-\frac{1}{y}\end{cases}}\) (*)

Ta có: \(A=\frac{x+y}{z}+\frac{x+z}{y}+\frac{y+z}{x}\)

\(=\frac{x}{z}+\frac{y}{z}+\frac{x}{y}+\frac{x}{y}+\frac{y}{x}+\frac{z}{x}\)

\(=\left(\frac{x}{z}+\frac{x}{y}\right)+\left(\frac{y}{x}+\frac{y}{z}\right)+\left(\frac{z}{x}+\frac{z}{y}\right)\)

\(=x\left(\frac{1}{z}+\frac{1}{y}\right)+y\left(\frac{1}{x}+\frac{1}{z}\right)+z\left(\frac{1}{x}+\frac{1}{y}\right)\)

Thay (*) vào,ta có : \(A=x.\left(\frac{-1}{x}\right)+y.\left(-\frac{1}{y}\right)+z.\left(-\frac{1}{z}\right)=\left(-1\right)+\left(-1\right)+\left(-1\right)=-3\)